Use Elimination for the answer on both :)

Use Elimination For The Answer On Both :)

Answers

Answer 1

The value of x and y after solving the equation -5x - 10y = 30 and - 3x + 3y = 27 by elimination method is -6 and 3 respectively.

What is the equation?

A formula known as an equation uses the equals sign to denote the equality of two expressions.

Given:

-5x - 10y = 30 and - 3x + 3y = 27

Solve by using the elimination method,

Divide the equation - 3x + 3y = 27 by 3 and equation -5x - 10y = 30 by 5 on both sides,

(- 3x + 3y) / 3 = 27 / 3  and (-5x - 10y) / 5 = 30 / 5

-x + y = 9  and -x - 2y = 6

Subtract equation

-x + y = 9 - (-x - 2y = 6)

-x + y - 9 = x - 2y - 6

y = 3

and - x + 3 = 9, x = -6

Therefore, the value of x and y after solving the equation -5x - 10y = 30 and - 3x + 3y = 27 by elimination method is -6 and 3 respectively.

To know more about equation:

https://brainly.com/question/12788590

#SPJ1


Related Questions

Suppose the amount of liquid dispensed by a certain machine is uniformly distributed with lower limit A = 8 oz and upper limit B = 10 oz. Describe how you would carry out simulation experiments to compare the sampling distribution of the (sample) fourth spread for sample sizes n = 5, 10, 20, and 30.

Answers

To compare the sampling distribution, first I would simulate the machine by randomly generating n number of samples between 8 and 10 oz with a uniform distribution and calculate the 4th spread for each sample size. I would then repeat this experiment multiple times and take the average of the fourth spread values for each sample size to compare the sampling distributions.

To compare the sampling distributions of the fourth spread for sample sizes n = 5, 10, 20, and 30, I would simulate the machine by randomly generating samples between 8 and 10 oz with a uniform distribution. For each sample size, I would calculate the fourth spread for each sample and then repeat the experiment multiple times. Finally, I would take the average of the fourth spread values for each sample size to compare the sampling distributions. This way, I would be able to see how the fourth spread changes as the sample size changes. It would also allow me to see how the fourth spread is affected by the uniform distribution of the machine's output.

To know more about sampling distribution refer to the link  brainly.com/question/13501743

#SPJ4

combine radicals/fractional exponents

Answers

The product of a,b and c from the fraction is 6/125

How to find product of numbers?

The product is the result of the multiplication of the numbers

From the given exponential fractions, we have

=(3x³y²/³)÷(125³*¹/³y³*¹/³

Simplifying the expression to have

(3x³y²/³)(125)(1)

⇒3/125x³y²/³

The expressed form is [tex]ax^{b} y^{c}[/tex]

a=3/125 b=3 c 2/3

Simplify further to have

⇒3/125*3/1*2/3

Simplify further to get your final answer as

=6/125

Therefore, the product of the fractional exponents is 6/125

Learn more about simplification of fractions on https://brainly.com/question/15605622

#SPJ1

3. suppose that a particular nfl team claims that the mean weight of their defensive players is 315 pounds. a rivaling team thinks that the mean weight of the defensive players on that team is different from 315 pounds they claim. suppose they sample 15 of the defensive players and find the players to have a mean weight of 300 pounds with sample standard deviation 8 pounds. set up an appropriate hypothesis test and make conclusions at significance level 0.08. assume weights are normally distributed.

Answers

We reject null hypothesis when hypothesis test, draw conclusions at significance level 0.08.

Given that,

Consider an NFL team that says the average weight of its defensive players is 315 pounds. A opposing team believes that their defensive players' average weight is lower than the 315 pounds they state. Consider that they take 15 defensive players as a sample and discover that the average weight of the players is 300 pounds, with an 8-pound sample standard deviation.

We have to create a suitable hypothesis test, draw conclusions at significance level 0.08, and then. Suppose that the weights are evenly distributed.

We know that,

Weights are normally distributed.

We are given μ = 315 , n= 15 , x = 300, s = 8, α = 0.08

The hypothesis are :

H₀ : μ = 315   v/s  H₁ : μ ≠ 315

The test statistic is given by,

z=x-μ/σ/√15

=300-315/8/√15

=-7.26

The critical value = z(α)  = z(0.04) = -1.75 and z(1-α)  = z(1-0.04)  = 1.75

Here calculated value of z > critical value of z

Therefore, we reject null hypothesis when hypothesis test, draw conclusions at significance level 0.08.

To learn more about hypothesis visit: https://brainly.com/question/15583625

#SPJ4

does anybody know how to answer this, and what I should look up to find out how to do this ​

Answers

Answer:

I think you have to look at the graph and find the anwser's to the problems below . ( I think )

Answer:

Below

Step-by-step explanation:

Domain is the set of values x can be for a function .....it looks like this one goes from -2 to 2 with asymptopes at these values:

-2< x < 2  

Range is the 'y' values a graph can have...this one goes from -1.5 -ish to 5.5 ish    

-1.5 <= y <= 5.5

Here is the graph with some lines on it:

Diego knows he will have a minimum of $2000 in his savings from the education award.Diego decides to get a part time job at night and on weekends to save additional money for college.

Inequality and meaning of variables



Graph

Answers

The money left is $5000 - $2000 = $3000 will be earned through an internship.

What is subtraction?

To subtract in mathematics is to take something away from a group or a number of objects.

In other meaning, subtraction is a mathematical operation such that two values are going to subtract and give a resultant value.

As per the given,

Money earned in award = $2000

Money had at the end of saving account = $5000

Money earned by internship = $5000 - $2000 = $3000

Hence "$3000 of the remaining $5000 will be earned through an internship".

For more about subtraction,

https://brainly.com/question/1927340

#SPJ1

a rectangle's length is 6 inches greater than its width. if the perimeter of the rectangle is 64 inches, find the length. (all answers are given in inches.)

Answers

The width of the rectangle is 6 inches and length of rectangle is 12 inches

Step-by-step explanation:

We have given the perimeter of the rectangle and the length of a rectangle is 6 inches longer than its width.

Let x be the width of the rectangle and x+6 be the length of the rectangle.

Perimeter = p = 36 inches

We have to find the values of width and length of rectangle.

The formula to find the Perimeter is:

p = 2l+2w

36 = 2(x+6)+2(x)

36 = 2x+12+2x

36 = 4x+12

36-12= 4x

24 = 4x

x = 6 inches.

Hence, the width of the rectangle is 6 inches and length of rectangle is 12 inches.

To know more about inches visit:

https://brainly.com/question/16311877

#SPJ4

The weight. In grams, of a population of bacteria at time t hours is modeled by the function W the solution to a logistic differential equation. Selected values of W and its first and second derivatives are shown in the table above. Which of the following statements is true? W (35 – W/), because the carrying capacity is 35 and the rate of change of the weight is 6 grams per hour when the weight is 10 grams. W (35 – W), because the carrying capacity is 35 and the fate of change of the weight is 3 grams per hour when the weight is 10 grams (70W). because the carrying capacity is 20 and the rate of change of the weight is 6 grams per hour when the weight is 10 grams. Om de operation because the cauruna capacity te 70 and the rate of change of the wengrana

Answers

With regards to the function model then the true statement as per first and second derivatives is: (C) dw/dt = 1/100 W  (70 - W),

When, W = 10 then dw/dt = 6

When W = 35 then d²w/d²t = 0

where the point influx occurs, the weight of the carrying capacity is half

Therefore, 35 = a/2

Then the carrying capacity (a) = 35 x 2

a = 70

A function's sensitivity to change with respect to a change in its argument is measured by the derivative of a function of a real variable. Calculus's core tool is the derivative. It is a crucial idea that is incredibly helpful in a variety of contexts: in daily life, the derivative can inform you how fast you are driving or assist you in predicting stock market changes; in machine learning, derivatives are crucial for function optimization.

Therefore, with regards to the function model then the true statement as per first and second derivatives is: (C) dw/dt = 1/100 W  (70 - W), because the carrying capacity is 70 and the rate of change of the weight is 6 grams per hour when the weight is 10 grams.

To learn more about derivatives refer here

brainly.com/question/23819325

#SPJ4

Read the following prompt and type your response in the space provided.
Karen is writing a word problem that can be represented by the equation 2x + 3 = 13.

A bucket weights 2 grams and balls are put into it that each weight 3 grams. The total weight of the bucket and balls is 13 grams. How many balls are in the bucket?

Does her word problem fit the equation? Justify your response..

Answers

No her word problem does not fit the equation. The equation has a starting, unchanging value of 3. But the word problem has a starting, unchanging value of 2 (the bucket weighs 2 grams).
The equation has an x coefficient of 2. But the word problem has a x coefficient of 3 (balls are put into [the bucket] that each weigh 3 grams), where x, the unknown variable, is the number of balls.
Although the numbers are the same, Karen switched the 2 and 3 values when going from the equation to the problem.

Answer:can you copy and paste it?

Step-by-step explanation:

Skylar and Wyatt both play soccer. Wyatt scored 2 times as many goals as Skylar. Together, they scored 15 goals. Could Skyler have scored 3 goals? Why or why not?

No, because 3(2) + 3 ≠ 15
No, because 3(2) ≠ 15
Yes, because 3 goals is less than the total number of goals scored
Yes, because 3 goals is less than the number of goals Wyatt scored

Answers

An equation is formed of two equal expressions. The correct option is A.What is an equation?An equation is formed when two equal expressions are equated together with the help of an equal sign '='.Let the goal scored by Skylar and Wyatt be x and y respectively. Now, given Wyatt scored 2 times as many goals as Skylar. Therefore,y = 2xAlso, Together they scored 15 goals. Therefore,x + y = 15x + 2x = 153x = 15x = 5Thus, Skylar and Wyatt have scored 5 and 10 goals respectively.Hence, the correct option is A.Learn more about Equation:brainly.com/question/2263981#SPJ2For more answers, Contact:
https://brainly.com/app/profile/63103549/answers

Answer:

The correct answer is A. No, because 3(2) + 3 ≠ 15.

Step-by-step explanation:

because the total number of goals scored by Skylar and Wyatt was 15, and if Skylar scored 3 goals, then Wyatt would have scored 3 * 2 = 6 goals. Since the total number of goals scored by both players is 15, this scenario is not possible. Therefore, Skylar could not have scored 3 goals.

m+ 1/3 < −2 1/4
Solve for m

Answers

To solve this inequality, we can start by converting the mixed numbers to improper fractions. The mixed numbers in this inequality are 1/3 and -2 1/4. The negative sign in front of 2 1/4 indicates that the resulting fraction will be negative. To convert 1/3 to an improper fraction, we can multiply the denominator (3) by the whole number (1) and add the result to the numerator (1). This gives us 1/3 = 4/3. To convert -2 1/4 to an improper fraction, we can multiply the denominator (4) by the whole number (-2) and add the result to the numerator (1). This gives us -2 1/4 = -9/4.

Next, we can combine the two fractions on the left side of the inequality by adding their numerators and denominators. This gives us m + 4/3 + -9/4. We can then simplify this expression by combining like terms. This gives us m - 9/12 + 4/3. We can further simplify this expression by adding the fractions with unlike denominators. To do this, we can find a common denominator by multiplying the two denominators together, which gives us 12 * 3 = 36. We can then convert the fractions to have this common denominator by multiplying the numerator and denominator of each fraction by the appropriate value. For m - 9/12 + 4/3, we have m - (9 * 3)/(12 * 3) + (4 * 12)/(3 * 12), which simplifies to m - 27/36 + 48/36. We can then combine like terms to get m + 21/36.

Finally, we can solve for m by moving all of the terms with m to one side of the inequality and all of the other terms to the other side. To do this, we can subtract 21/36 from both sides of the inequality, which gives us m + 21/36 - 21/36 < -2 1/4 - 21/36. This simplifies to m < -2 1/4 - 21/36, which we can further simplify by converting -2 1/4 - 21/36 to an improper fraction. To do this, we can multiply the denominator (4) by the whole number (-2) and add the result to the numerator (1), which gives us -2 1/4 = -9/4. We can then add -9/4 and -21/36 to get -9/4 - 21/36 = -105/36. Finally, we can convert this to a mixed number by dividing the numerator (-105) by the denominator (36) and taking the quotient (-2) as the whole number and the remainder (27) as the numerator of the fraction. This gives us -105/36 =

probability that point estimate is within population mean with standard error and population standard deviation

Answers

The probability is 0.7888.

Standard error of the mean [tex]$\sigma_{\bar{x}}=20$[/tex]

The population standard deviation (σ)=500

Standard deviation of [tex]$\bar{x}$[/tex] is given by:

[tex]\sigma_{\bar{x}} & =\frac{\sigma}{\sqrt{n}} \\\\\Rightarrow 20 & =\frac{500}{\sqrt{\mathrm{n}}} \\\\\Rightarrow n & =\left(\frac{500}{20}\right)^2 \\[/tex]=625

Probability that the point estimate [tex]$\bar{x}$[/tex] was within ±25 of the population mean is: [tex]$$p(\mu-25 < \bar{x} < \mu+25)$$[/tex]

At [tex]$\bar{x}[/tex]=μ-25, we have

[tex]z & =\frac{-25}{20} \\[/tex]=-1.25

Z=-1.25

At [tex]$\bar{x}[/tex]=μ+25, we have

[tex]z & =\frac{25}{20} \\[/tex]=1.25

Z=1.25

So the required probability is:

[tex]$$\begin{aligned}p(\mu-25 < \bar{x} < \mu+25) & =p(-1.25 < z < 1.25) \\& =p(z < 1.25)-p(z < -1.25) \\\end{aligned}$$[/tex]

                                     =0.8944-0.1056

                                     =0.7888

Therefore, probability that point estimate is within ±25 population mean is 0.7888.

For more such questions about Probability

https://brainly.com/question/27992935

#SPJ4

A researcher reports survey results by stating that the standard error of the mean is 20 . The population standard deviation is 500 . How large was the sample used in this survey

a. What is the probability that the point estimate was within ±25 of the population mean?

data are collected on the 35 students in a college history course. which of the following is not a variable for the data set? responses student birth month student birth month political affiliation of student political affiliation of student student age student age student address

Answers

The correct statement is-number of students in the data set

In research and data collecting, something that is being measured and whose values can change is referred to as a variable.

Given that we have a range of alternatives from January to December, the aforementioned example demonstrates how the student birth month is a variable. The student's ability to indicate whether they support particular political parties or not makes their political affiliation another variable. Since they are college students, the age of the student is another variable, with replies falling within a range of 20 to 25.

Since students will give different answers, whether they live at the same address or a different one, student address is also a variable.

Know more about Data at;

https://brainly.com/question/26711803

#SPJ4

If a sample mean is 32, which of the following is most likely the range of possible values that best describes an estimate for the population mean? A. (28,36) B. (34,42) C. (32, 42) D. (30,38)​

Answers

Answer:

Step-by-step explanation:

If a sample mean is 32, the range of possible values that best describes an estimate for the population mean is most likely (30, 38).

The sample mean is a statistic that is used to estimate the population mean. In general, the sample mean is a good estimate of the population mean, but it is not always perfectly accurate. There is always some degree of uncertainty or variability associated with any estimate, and this is especially true when the sample size is small.

One way to quantify this uncertainty is to use a confidence interval, which is a range of values that is likely to contain the population mean with a certain level of confidence. For example, a 95% confidence interval is a range of values that is likely to contain the population mean with 95% confidence.

In this case, if a sample mean is 32, a 95% confidence interval for the population mean would likely be a range of values centered around 32 and extending approximately 2 standard errors in either direction. For a sample mean of 32, a range of values extending 2 standard errors in either direction would be approximately (30, 38). Therefore, the range of possible values that best describes an estimate for the population mean is most likely (30, 38).

If a sample mean is 32, (30, 38) is most likely the range of possible values that best describes an estimate for the population mean. the correct option is option D.

What is range?

Range serves as a statistical measure of dispersion in mathematics, or how widely spaced a particular data collection is from smallest to biggest. The range in a piece of data is the distinction between the highest and lowest number. The confidence interval of 95% for the population mean in this scenario, assuming the sample mean is 32, is likely to include a range of values centred around 32.

Extending around 2 standard errors in each direction. A range of results spanning two standard deviations in either way would result in around (30, 38) for a sample mean of 32. Thus, the range of potential values that most accurately captures an estimate of the population mean is (30, 38).

Therefore, the correct option is option D.

To know more about range, here:

https://brainly.com/question/30821383

#SPJ5

ASAP HELP!! Which one is equivalent I think option D but I don’t know!!

Answers

Answer:

D

Step-by-step explanation:

6^(4+0) = 6^4

6^(-10) : 6^4

6^(-10-4)

6^(-14)

1/6^(14)

Write 58,000 in scientific notation by filling in the blanks of the structure

Answers

Answer:  [tex]5.8\times 10^4[/tex]

Reason:

Place the decimal point between the 5 and the 8. So we have the value 5.8

To get back to 58000, we need to move the decimal point 4 spots to the right. This is why we have 4 as the exponent over 10.

All scientific notation values are of the form [tex]a \times 10^b[/tex]

where [tex]1 \le a < 10[/tex] or [tex]-10 < a \le 1[/tex]

The b value is an integer.

A medical researcher wishes to see if he can lower the cholesterol levels through diet in 6 people by showing a film about the effects of high cholesterol levels. The data are shown. At =α0.05 , did the cholesterol level decrease on average? State the hypotheses and identify the claim with the correct hypothesis. H0 : ▼(Choose one) H1 : ▼(Choose one) This hypothesis test is a ▼(Choose one) test. Compute the test value. Always round t score values to three decimal places. t= Find the interval for the P -value. The correct interval is Make the decision. ▼(Choose one) the null hypothesis. Summarize the results. There is ▼(Choose one) that the mean heights are different.

Answers

For the study about cholestrol levels of people was taken by medical researcher.

a) Null and Alternative hypothesis for study is

H₀ : μd ≥0

Hₐ : μd <0

where μd --> average

b) The value of t-statistic is 3.78118

c) The critical value for t-test is - 2.015.

d) We fail to reject null hypothesis, H₀

e) There is sufficient evidence to support the claim of researcher that cholesterol level decreases on average.

The above table represents the patients and their cholesterol levels before and after they showed the film about the effect of high cholesterol levels. The "d" column shows the difference in their cholesterol levels.

Singnificance level, α =0.05

∑d = 122, n = 6 , ∑d² = 3348

Xd -bar = 122/6 = 20.33

Standard deviations, σ = √(∑d² - ∑d/n)/(n-1)

σ = √(3348 - 20.33)/5 = 13.170

a) The Null and Alternative hypothesis are

H₀ : μd ≥0

Hₐ : μd <0

b) test- statistic,

t - value = (Xd-bar - μd)/σ /√n

=> t = (20.33 - 0)/13.17/√6

=> t = 20.33/5.37662

=> t = 3.78118

c) degree of freedom, df = n-1 = 5

The critical t-value for a given confidence level c and sample size n is obtained by computing the quantity tα/2 for t-distribution with (n-1) degrees of freedom.

α = 0.05 , α/2 = 0.25 , df = 5

t(₀.₂₅, ₅) = - 2.015

d) Decision: As we seen t-value= 3.781 and t-critical value = - 2.015

so, t -value > critical value. Therefore, we fail to reject null hypothesis.

e) Conclusion: There is sufficient evidence to claim that the cholesterol level decrease on average.

Hence, we got all required answers.

To learn more about Hypothesis testing, refer:

https://brainly.com/question/4232174

#SPJ4

Complete question:

Cholesterol Levels A medical researcher wishes to see if he can lower the cholesterol levels through diet in 6 people by showing a film about the effects of high cholesterol levels. The data are shown. At =α0.05 , did the cholesterol level decrease on average? Use the P -value method and tables. Patient 1 2 3 4 5 6 Before 250 219 218 224 209 223 After 216 203 192 190 207 213 State the hypotheses and identify the claim with the correct hypothesis. H0 : ▼(Choose one) H1 : ▼(Choose one) This hypothesis test is a ▼(Choose one) test. Compute the test value. Always round t score values to three decimal places. t= Find the interval for the P -value. The correct interval is Make the decision. ▼(Choose one) the null hypothesis. Summarize the results. There is ▼(Choose one) that the mean heights are different

The sum of 6 and the squared of 12 numerical expression

Answers

6 + 12^2

12 squared would be 12 to the second power

since the directions say "the sum"

you would use addition to add 6 and 12^2

13) The pet shop has 6,157 dog treats. They packed the treats into bags with 7 pieces in each bag. Any treats
that did not fit into a bag of 7 were given to the groomer to use to reward animals getting bathed. How many
bags of treats did they fill? How many treats were given to the groomer?

bags of treats and gave
left over treats to the groomer.
Answer: The pet shop filled___bags of treats and gave ____left over treats to the groomer.

Answers

The number of bags filled by the pet shop is 879 and the number of treats they gave to the groomer is 4.

What are arithmetic operations?

The four basic operations of arithmetic can be used to add, subtract, multiply, or divide two or even more quantities.

They cover topics like the study of integers and the order of operations, which are relevant to all other areas of mathematics including algebra, data processing, and geometry.

As per the data given in the question,

Total number of dog treats in the shop = 6157

Number of treats per bag = 7

Extra treats will be given to the groomer.

Then, the number of bags will be,

6157/7 = 879.57

This means that a total of 879 bags are there, with 7 pieces of treats per bag.

So, the total number of treats in 879 bags will be,

879 × 7 = 6153

So, the amount of treats left for the groomer is,

6157 - 6153 = 4 treats.

To know more about arithmetic operations:

https://brainly.com/question/13585407

#SPJ1

use the fundamental theorem to evaluate the definite integral exactly. 16 1 3/pi of x dx enter the exact answer. g

Answers

The exact answer to the definite integral ∫16^1^3/π of x dx using the fundamental theorem is 8/π^2.

The fundamental theorem of calculus states that if f(x) is a continuous function on the interval [a, b] and F(x) is an antiderivative of f(x), then:

∫a^bf(x)dx = F(b) - F(a)

To use the fundamental theorem to evaluate the definite integral ∫16^1^3/π of x dx, we need to find an antiderivative of x. The antiderivative of x is x^2/2 + C, where C is an arbitrary constant.

Therefore, the definite integral ∫16^1^3/π of x dx can be written as:

∫16^1^3/π x dx = (x^2/2 + C) |16^1^3/π

Evaluating this expression gives us:

(3/π)^2/2 + C - (1/π)^2/2 + C = (9/π^2 - 1/π^2)/2 + 2C

Since C is an arbitrary constant, it cancels out when we take the difference between the upper and lower limits of integration. Therefore, the definite integral ∫16^1^3/π of x dx is equal to:

(9/π^2 - 1/π^2)/2 = 8/π^2

Learn more about Integration at:

brainly.com/question/27419605

#SPJ4

write an equation that describes the sequence below. use n to represent the position of a term in the sequence

Answers

Although part of your question is missing, you might be referring to this full question: Write an expression that describe the sequence below. Use n to represent the position of a term in the sequence, where n = 1 for the first term. –1, 0, 1, 2, ...

The expression that describes the given sequence is: aₙ = –2 + n

This is an arithmetic sequence. Arithmetic sequence is a sequence of numbers in which every term (except the first term) is provided by adding a constant number to the previous term.

What is the formula used for arithmetic sequence?

An arithmetic sequence can be defined by an explicit formula in which aₙ = d (n – 1) + c, where d is the common difference between consecutive terms, and c = a₁.

In this case, the given sequence is –1, 0, 1, 2, ...

This is an arithmetic sequence with common difference.

Because of:

0 – (–1) = 1

1 – 0 = 1

2 – 1 = 1 …

Then, a₁ = –1

Now, substitute the value of a₁ = –1, d = 1 into the formula.

aₙ = d (n – 1) + c

aₙ = 1 (n – 1) + (–1)

aₙ = n + (–2)

aₙ = –2 + n

Hence, the expression that describes the given sequence is: aₙ = –2 + n.

Learn more about arithmetic sequence at: https://brainly.com/question/15412619

#SPJ4

can someone help with this question please? no one still hasn’t answered it and i’m trying to help someone out by posting it for them

Answers

Answer:

6*3 + 6*2

6*5

6(3+2)

Step-by-step explanation:

All of these equations lead to the same answer which is 30.

The two diagrams are basically the same but they are just separated apart. When you look for the area of both diagrams, it leads to the same answer.

Answer:

Step6x3 = 18. And. 2x6 = 12. Then you add the amounts, 18+12 = 30-by-step explanation:

2)
a: b is 2:5 and b: c is 3:8
Work out a:c
Give your answer in its simplest form.

Answers

Answer:

a : c = 3 : 20

Step-by-step explanation:

Given ratios:

a : b = 2 : 5b : c = 3 : 8

The value of b is 3 and 5.

The lowest common multiple of 3 and 5 is 15.

Multiply each ratio by a factor of 15 so that b is 15.

Multiply ratio a : b by 3:

⇒ a : b = 6 : 15

Multiply ratio b : c by 5:

⇒ b : c = 15 : 40

Therefore:

⇒ a : b : c = 6 : 15 : 40

So a : c is:

⇒ a : c = 6 : 40

Simplest form:

⇒ a : c = 3 : 20

If two lines are perpendicular, then the lines intersect to form four right angles that are 180º each.

True

False

Answers

Answer:

False.  They would form 4 right angles (90)

Step-by-step explanation:

Right angles are always 90 degrees, so false.


Enter the ordered pair for the vertices for r(90°, s)(QRST).

I need help with this please help me

Answers

The ordered pair for the vertices for r(90°, s)(QRST) include the following:

Q' = (-5, 1)

R' = (-3, -1)

S' = (0, 0)

T' = (-3, -2)

What is a rotation?

In Geometry, the rotation of a point 90° about the center (origin) in a counterclockwise (anticlockwise) direction would produce a point that has these coordinates (-y, x).

Note: The point S represents the center (origin) of this geometric figure.

By applying a rotation of 90° counterclockwise to the vertices for r, the coordinates of the vertices of the image point r' are as follows:

(x, y)                               →            (-y, x)

Ordered pair Q  = (1, 5) → Ordered pair Q' = (-5, 1)

Ordered pair R = (-1, 3) → Ordered pair R' = (-3, -1)

Ordered pair S = (0, 0) → Ordered pair S' = (0, 0)

Ordered pair T = (-2, 3) → Ordered pair T' = (-3, -2)

Read more on vertices here: https://brainly.com/question/24600877

#SPJ1

Which table of constant of proportionality between y and x of 8/5?

Answers

The table with constant of proportionality between x and y is table A

What is constant of proportionality?

We should be able to know that constant of proportionality shows a type of relationship between two variables that shows a constant relationship or same ratio.

The given fraction is 8/5

Rewrite 8/5 as a ratio of two numbers

8:5

Constant of proportionality is written as

y=k/x

Making k the subject, k=y/x

Where k is a constant.

From table A,

The constant  (K) is 4/1 = 4:1

Now multiply 4*8=32

8*1=5

Therefore 32/5 is a proportion of 8/5

Also using the constant k=10

⇒10:1   Then 10*8=80

And 1*5=5

⇒ 80/5=16

Also in the last constant K=11, we have 11:1

⇒[tex]\frac{11*8}{1*5}[/tex] =88/5

In conclusion option A gives the table that has constant of proportionality to 8/5

Read more about constant of proportionality on https://brainly.com/question/29126727

#SPJ1

The integer 5 makes which of the following equation false

Answers

Answer: 3m+4=6m is the false equation

Step-by-step explanation:

Substitute 5 into the 4 equations to find whether they are true or false

3(5)+4=6(5)

15+4=30

19[tex]\neq[/tex]30 False

-5(5-7)=10

-5(-2)=10

10=10 true

17=4(5)-3

17=20-3

17=17 True

5+9=14

14=14 True

3m+4=6m is the false equation

three fourths times the quantity x plus 6 end quantity minus one fourth times x equals two fourths times the quantity x plus 9 end quantity

Answers

The value of x in the equation, 3/4x + 6 - 1/4x = 2/4x + 9, is: x = -2.

How to Solve an Equation?

Solving an equation involves finding the value of the variable in the equation. The value, when plugged into the equation would make it true.

The equation given is, 3/4x + 6 - 1/4x = 2/4x + 9, to solve for the value of x, follow the steps below:

3/4x + 6 - 1/4x = 2/4x + 9 [given]

Combine like terms:

3/4x - 2/4x - 1/4x = 9 - 6

(3 - 2 - 1)/4x = 3

-6/4x = 3

Simplify

-3/2x = 3

Multiply both sides by -2/3:

-3/2x × -2/3 = 3 × -2/3

x = -6/3

x = -2

Learn more about equations on:

https://brainly.com/question/25678139

#SPJ1

Margin of error: 0.04; confidence level: 95%; from a prior study, p is estimated by the decimal equivalent of 60%
A) 577 B) 1441 C) 996 D) 519

Answers

The required sample size of the given value is 577.

What is margin of error?

In a random survey sample, a margin of error is a statistical measurement that takes into account the discrepancy between actual and anticipated findings. Simply said, you may determine the degree of unpredictability in data and research results using the margin of error.

The formula to calculate the sample size  :-

n = (p(1-p)((Zα/₂)/E)²

Given :  Estimated proportion  p = 0.60

Margin of error : E = 0.04

Significance level : α = 1-0.95 = 0.05

Critical value : Zα/₂ = 1.96

Now, the required sample size will be :-

n = (0.60)(0.11)(1.96/0.04)²

= 6.6 * (0.0784)²

= 0.57744

Hence, the required minimum sample size of the given is 577.

To know more about margin of error check the below link:

https://brainly.com/question/24289590

#SPJ1

omplete the proof to show that abcd is a parallelogram. on a coordinate plane, quadrilateral a b c d is shown. point a is at (negative 2, negative 2), point b is at (negative 3, 4), point c is at (2, 2), and point d is at (3, negative 4). the slope of line segment b c is startfraction 4 minus 2 over negative 3 minus 2 endfraction

Answers

The quadrilateral abcd is a parallelogram because the slopes of opposite sides are equal.

In the given figure we have a quadrilateral  ABCD with A (-2,-2) , B(-3,4), C(2,2)

and D(3,-4).

We know that when the slope of two lines are equal then they are parallel.

In the given figure, Slope of BC= Slope of AD =

⇒ BC and AD are parallel .

Also, Slope of CD= Slope of BA =-6

⇒ CD and BA are parallel .

ABCD is a parallelogram because both pairs of opposite sides are parallel.

Hence, the correct reason for the given space is "slopes of opposite sides are equal"

To learn more about quadrilaterals refre here

https://brainly.com/question/23935806

#SPJ4

A survey of 20 randomly selected adult men showed that the mean time they spend per week watching sports on television is 10.53 hours with a standard deviation of 1.32 hours. Assuming that the time spent per week watching sports on television by all adult men is (approximately) normally distributed, construct a 90% confidence interval for the population mean,μ . Round your answers to two decimal places. Lower bound: Upper bound:

Answers

For a 90% confidence interval the upper and lower bound is 11.50 and 9.56 respectively.

What is confidence interval?

Confidence interval means the probability that a population parameter will fall between a set of values.

What is the formula of confidence interval?

The formula for confidence interval is C.I = u ± z (sd/[tex]\sqrt{n\\}[/tex] ) where u is the mean, sd is the standard deviation and z is the confidence level value. For an area of 90% around the mean, which makes 45% on each side, for the confidence level we will find the z-score at 0.45+0.5 = 0.95 area, and then double it since we need for both sides. At 0.95, the z-score is approximately 1.64. Thus, z is 3.28.

Plugging the values in the formula for C.I gives the upper bound as 10.53 + 0.968 = 11.50 and the lower bound as 10.53 - 0.968 = 9.56

To learn more about confidence interval visit the link below:

https://brainly.com/question/24131141

#SBJ4

Other Questions
suppose list1 is an myarraylist and list2 is a mylinkedlist. both contains 1 million double values. analyze the following code: a: for (int i 5. The volcanic eruption of Mt. St. Helens in the state of Washington in 1980produced a considerable quantity of a radioactive element in a gaseousstate. Atoms in the radioactive substance had 86 protons. Which was it?RubidiumBrominePoloniumRadon technicians are paid a fixed amount of hours for a job based on an established average time to perform the repair is referred to as . a cloud-to-ground lightning discharge will sometimes appear to flicker. this is because: question 5 multiple choice options you are able to see the separate steps of the stepped leader you are able to distinquish separate return strokes the bright light causes you to blink of refraction caused by turbulent thunderstorm winds If angle EBC is congruent to angle ECB, then ? is congruent to ? Haley B. Workin did eight chores in the morning. She did 40% of her daily chores in the morning. How many daily chores does she have? the nurse is reviewing the laboratory results for a client. which laboratory findings would the nurse correlate with nephrotic syndrome? Suppose that FAR is similar to SUN. Part A: Identify the corresponding angles. When Irish immigrants came to the United States, many attempted to learn the traits and behaviors of the dominant white culture. What does this behavior by the Irish immigrants BEST represent? A. pluralism B. segregation C. assimilation D. amalgamation Nathan is going to drive from his house to City A without stopping. Let D representNathan's distance from City At hours after leaving his house. A graph of D is shownbelow. Write an equation for D then state the slope of the graph and determine itsinterpretation in the context of the problem. the evolutionary mechanism that allows for species to adapt to the environmental conditions found in their habitat is..... I need help on this Math. You have to find the y-intercept, then put it he y-intercept in coordinate form, and the slope. It was due so long ago. I don't know what to do. How many grams of CaCl would be required to produce a 3.5 M solution with a volumeof 2.0 L? FILL IN THE BLANK liam's bmi is 33.0. according to his food records, he consumes an average of 3000 kcal/day. he would like to lose 4 pounds in 4 weeks by reducing his caloric intake. based on this information, liam should consume an average of______kcal/day. according to okin, the fact that more women are working outside the home (and getting paid more for it) Graph the points (0.5,4.5) and (-0.5,-4.5) an economy is operating at long run equilibrium. what is most likely the result of agggresive exspansionary fiscal policy You buy the following from Amazon:25 flash drives for $10 eachWhat is the Subtotal: $Today only they are 25% off, how much do you save: $What is the new Subtotal: $Sales tax is 4%: $What is the total: $(For all numbers, put in the change. If there is no change, make sure you put .00. For example, if the amount was 27, make sure you put 27.00.If there is no number in the pennies place, make sure you put a 0. For example, if your amount was 5.4, make sure you put 5.40) highly efficient two-step synthesis of 2,5-furandicarboxylic acid from fructose without 5-hydroxymethylfurfural (hmf) separation: in situ oxidation of hmf in alkaline aqueous h2o/dmso mixed solvent under mild conditions 30 october 2018 Which of the following are correct statements about the Internal rate of return? (Check the apply.) Check all that apply. A) IRR is expressed in dollars. B) If a project has a positive NPV, then the IRR is less than the hurdle rate. C) IRR reflects the time value of money. D) IRR uses accrual income as the measurement basis. E) The higher the IRR, the better.